Bangladesh IMO TST 1: 2011/ NT-1 (P 3)

For discussing Olympiad Level Number Theory problems
User avatar
Moon
Site Admin
Posts:751
Joined:Tue Nov 02, 2010 7:52 pm
Location:Dhaka, Bangladesh
Contact:
Bangladesh IMO TST 1: 2011/ NT-1 (P 3)

Unread post by Moon » Sun Mar 13, 2011 7:55 am

Problem 3:
The integer $x$ is at least $3$ and $n=x^6-1$. Let $p$ be a prime and $k$ be a positive integer such that $p^k$ is a factor of $n$. Show that $p^{3k}<8n$.
"Inspiration is needed in geometry, just as much as in poetry." -- Aleksandr Pushkin

Please install LaTeX fonts in your PC for better looking equations,
learn how to write equations, and don't forget to read Forum Guide and Rules.

User avatar
nayel
Posts:268
Joined:Tue Dec 07, 2010 7:38 pm
Location:Dhaka, Bangladesh or Cambridge, UK

Re: Bangladesh IMO TST 1: 2011/ NT-1 (P 3)

Unread post by nayel » Mon Mar 14, 2011 12:35 am

Let $k$ be the largest positive integer such that $p^k\mid n$. Then it suffices to prove that $p^{3k}<8n$. First assume that $p>3$.

Claim. $p^k<2x^2$.
Proof. We have $p^k\mid x^6-1=(x^3-1)(x^3+1)$. Hence $p^k\mid x^3-1$ or $p^k\mid x^3+1$. Note that $p$ cannot divide both $x^3-1$ and $x^3+1$ as that would imply $p\mid -(x^3-1)+(x^3+1)=2$ , impossible by our choice of $p$.
Case 1. $p^k\mid x^3-1$.
This implies $p^k\mid (x-1)(x^2+x+1)$. If $p$ divides both $x-1$ and $x^2+x+1$, then $p\mid (x^2+x+1)-(x+2)(x-1)=3$ , impossible since $p>3$. Thus $p^k\le\max\{x-1,x^2+x+1\}<2x^2$ for $x\ge 3$. Thus our claim holds in this case.
Case 2. $p^k\mid x^3+1$.
Then we have $p^k\mid (x+1)(x^2-x+1)$. If $p$ divides both $x+1$ and $x^2-x+1$, then $p\mid (x^2-x+1)-(x-2)(x+1)=3$ , impossible since $p>3$. Thus $p^k\le\max\{x+1,x^2-x+1\}<2x^2$ for $x\ge 3$. Therefore the claim holds in all cases.

Therefore $p^k\le 2x^2-1$ which implies $p^{3k}<(2x^2)(2x^2)(2x^2-1)=8x^6-4x^4<8x^6-8=8n$, as required.

Now we have to examine the case $p\le 3$. I'll post the rest of the solution a few moments later.
"Everything should be made as simple as possible, but not simpler." - Albert Einstein

User avatar
nayel
Posts:268
Joined:Tue Dec 07, 2010 7:38 pm
Location:Dhaka, Bangladesh or Cambridge, UK

Re: Bangladesh IMO TST 1: 2011/ NT-1 (P 3)

Unread post by nayel » Mon Mar 14, 2011 1:09 am

(Remaining part of the previous solution)
i) $p=2$: Then $2^k\mid x^6-1$, so $x$ is odd. But $x^2\pm x+1$ is odd, so $2^k\mid x^6-1=(x-1)(x+1)(x^2-x+1)(x^2+x+1)\Rightarrow 2^k\mid (x-1)(x+1)=x^2-1$. Therefore $p^{3k}=2^{3k}\mid (x^2-1)^3<8x^6-8=8n$, as required.

ii) $p=3$: Then $3^k\mid x^6-1=(x^2-1)(x^4+x^2+1)$.
Lemma. For any prime $p$ and positive integer $a$, $\displaystyle\gcd\left(a-1,\frac{a^p-1}{a-1}\right)\in\{1,p\}$.
Proof. Let $g$ be the gcd in the statement. Then $g\mid a-1$, so $a\equiv 1\pmod g$ and hence $\displaystyle\frac{a^p-1}{a-1}=a^{p-1}+a^{p-2}+\dots+1\equiv p\pmod g$. Therefore $g\mid p\Rightarrow g=1$ or $p$. $\blacksquare$
Hence it follows that $\gcd(x^2-1,x^4+x^2+1)=1$ or $3$. In the first case, $x$ must be divisible by $3$, which is impossible. Hence $\gcd(x^2-1,x^4+x^2+1)=3$. This implies that $3$ divides one of $x^2-1,x^4+x^2+1$ and $3^{k-1}$ divides the other.

i) If $3^{k-1}\mid x^2-1$, then $3^{k-1}\mid x-1$ or $x+1$ which implies $3^{k-1}\le x+1$. Hence $3^{3k-3}\le (x+1)^3\Rightarrow 3^{3k}\le (3x+3)^3<8(x^6-1)=8n$, which holds for all $x\ge 3$ by induction.

ii) If $3^{k-1}\mid x^4+x^2+1$ then $3\mid x^2-1$ and $3^2\nmid x^2-1$. Hence $x^2-1\equiv 3$ or $6\pmod{3^2}$, which implies $x^4+x^2+1\equiv 12\pmod{3^2}$. Thus $k-1=1\Rightarrow k=2$, and therefore $3^{3k}=3^6<8(x^6-1)=8n$. This completes the solution.
"Everything should be made as simple as possible, but not simpler." - Albert Einstein

User avatar
Masum
Posts:592
Joined:Tue Dec 07, 2010 1:12 pm
Location:Dhaka,Bangladesh

Re: Bangladesh IMO TST 1: 2011/ NT-1 (P 3)

Unread post by Masum » Tue Mar 22, 2011 3:51 pm

Sorry for my late reply.Here is the complete solution.But it is actually not different from the previous,only in using LTE.
Lemma 1: $gcd(x^2-x+1,x^2+x+1)=1$
Proof:
Let $gcd (x^2-x+1,x^2+x+1)=g$
Then $g|x^2-x+1,x^2+x+1$ so their difference $2x$ too.But note that $g$ odd.So $g|x|x^2-x,x^2-x+1$,so their difference $1$ too.That means that $g=1$
Lemma 2: $gcd(x-1,\frac {x^n-1} {x-1})|n$
$x^n-1=(x-1)(x^{n-1}+...+1)$
From the Remainder Theorem,$gcd(x-1,x^{n-1}+...+1)=gcd(x-1,1+....+1)=gcd(x-1,n)$ which of-course divide $n$.
When $n=p$ a prime,we have $gcd$ is $1$ or $p$.
Lemma 3:
$8(x^3-1)>27(x+1)$
Proof:
Straight forward if we use Induction.
We consider three cases:
Case 1:$p=2$ and let $v_{2}(n)=k$
Note that $x^2-x+1$ and $x^2+x+1$ both are odd.So then if $x$ even,it is trivial.Let's see when $x$ odd.
Again note that if $v_2(x+1)=l$ or $v_2(x-1)=l, v_2(x^2-1)=l+1$.So then $l+1=k$ and $x^2-1\ge 2^k$
Almost obviously $2(x^2+x+1)>x^2-1\ge 2^k$ and $4(x^2-x+1)>x^2-1\ge 2^k$
Thus $8(x^2-1)(x^2+x+1)(x^2-x+1)>2^k.2^k.2^k\Longrightarrow 2^{3k}$
When $p$ odd,$p|x^3+1$ or $x^3-1$,otherwise $p$ would divide their difference $2$.
Case 2:$p=3$
We can assume $x>4$ after checking this true for $x=3,4$
Let $3|x+1,$then the case $3|x-1$ would be much similar to that.
Let $v_3(x+1)=l,l>1$ since the theorem is obviously valid for $l=1$.Then according to LTE $v_3(x^3+1)=l+1$ or $l+1=k$
Also $3\not| \frac {x^2-x+1} {3}$ because if $3^2|x^2-x+1$,then $gcd(x^2-x+1,x+1)=9$ at least,contradicting the fact that $gcd(x+1,x^2-x+1)=3$(we get it from the lemma $2$)
Again note that $x^2+x+1>3(x+1)$ since $x\ge 3$ is given.Then $x^2-x+1>3.3^l=3^k$
From lemma $3,$we may conclude that $8(x^3-1)>27(x+1)\ge 27.3^l=27.3^{k-1}$
$x+1\ge 3^l=3^{k-1}$
Multiplying the inequalities,we get $8(x^3-1)(x+1)(x^2-x+1)>27.(3^{k-1})^3=3^{3k}$
Case 3: We shall see the case for $x+1$,the other case is similar.
Now $p\not| x+1,$so $p^k|x^2-x+1$ and then $x^2-x+1\ge p^k$
Then almost obviously $4(x^2-1)>x^2-x+1\ge p^k$ and $2(x^2+x+1)>p^k$
Multiply them and the result follows.
One one thing is neutral in the universe, that is $0$.

Post Reply